some confusion about supremum and infimum












3












$begingroup$



Let $a_{mn}$ be a double array or real numbers. Define $$A=liminf_{nrightarrow infty} limsup_{mrightarrowinfty}a_{mn}\B=limsup_{nrightarrow infty} liminf_{mrightarrowinfty}a_{mn},$$



Then which one is true?




  1. $Ale B$

  2. $Age B $




My attempt :



I think option $1$ that $Ale B$ is true because the supremum is always greater than then infimum.



Is this correct ? Any hints/solution would be appreciated.










share|cite|improve this question











$endgroup$

















    3












    $begingroup$



    Let $a_{mn}$ be a double array or real numbers. Define $$A=liminf_{nrightarrow infty} limsup_{mrightarrowinfty}a_{mn}\B=limsup_{nrightarrow infty} liminf_{mrightarrowinfty}a_{mn},$$



    Then which one is true?




    1. $Ale B$

    2. $Age B $




    My attempt :



    I think option $1$ that $Ale B$ is true because the supremum is always greater than then infimum.



    Is this correct ? Any hints/solution would be appreciated.










    share|cite|improve this question











    $endgroup$















      3












      3








      3





      $begingroup$



      Let $a_{mn}$ be a double array or real numbers. Define $$A=liminf_{nrightarrow infty} limsup_{mrightarrowinfty}a_{mn}\B=limsup_{nrightarrow infty} liminf_{mrightarrowinfty}a_{mn},$$



      Then which one is true?




      1. $Ale B$

      2. $Age B $




      My attempt :



      I think option $1$ that $Ale B$ is true because the supremum is always greater than then infimum.



      Is this correct ? Any hints/solution would be appreciated.










      share|cite|improve this question











      $endgroup$





      Let $a_{mn}$ be a double array or real numbers. Define $$A=liminf_{nrightarrow infty} limsup_{mrightarrowinfty}a_{mn}\B=limsup_{nrightarrow infty} liminf_{mrightarrowinfty}a_{mn},$$



      Then which one is true?




      1. $Ale B$

      2. $Age B $




      My attempt :



      I think option $1$ that $Ale B$ is true because the supremum is always greater than then infimum.



      Is this correct ? Any hints/solution would be appreciated.







      real-analysis






      share|cite|improve this question















      share|cite|improve this question













      share|cite|improve this question




      share|cite|improve this question








      edited Dec 15 '18 at 7:50









      Jonathan

      16312




      16312










      asked Dec 15 '18 at 6:02









      jasminejasmine

      1,781417




      1,781417






















          1 Answer
          1






          active

          oldest

          votes


















          6












          $begingroup$

          Of course the supremum is always greater than or equal to the infimum. However, if one has mixtures of $limsup$ and $liminf$, it is indecisive which one is greater. For example, let $$
          a_{mn} = (-1)^n (1-frac{1}{2^m}).
          $$
          Then we have
          $$
          1=liminf_{mtoinfty}limsup_{ntoinfty}a_{mn} >limsup_{mtoinfty}liminf_{ntoinfty}a_{mn} =-1
          $$
          while
          $$
          -1=liminf_{ntoinfty}limsup_{mtoinfty}a_{mn}<limsup_{ntoinfty}liminf_{mtoinfty}a_{mn}=1.
          $$






          share|cite|improve this answer









          $endgroup$













            Your Answer





            StackExchange.ifUsing("editor", function () {
            return StackExchange.using("mathjaxEditing", function () {
            StackExchange.MarkdownEditor.creationCallbacks.add(function (editor, postfix) {
            StackExchange.mathjaxEditing.prepareWmdForMathJax(editor, postfix, [["$", "$"], ["\\(","\\)"]]);
            });
            });
            }, "mathjax-editing");

            StackExchange.ready(function() {
            var channelOptions = {
            tags: "".split(" "),
            id: "69"
            };
            initTagRenderer("".split(" "), "".split(" "), channelOptions);

            StackExchange.using("externalEditor", function() {
            // Have to fire editor after snippets, if snippets enabled
            if (StackExchange.settings.snippets.snippetsEnabled) {
            StackExchange.using("snippets", function() {
            createEditor();
            });
            }
            else {
            createEditor();
            }
            });

            function createEditor() {
            StackExchange.prepareEditor({
            heartbeatType: 'answer',
            autoActivateHeartbeat: false,
            convertImagesToLinks: true,
            noModals: true,
            showLowRepImageUploadWarning: true,
            reputationToPostImages: 10,
            bindNavPrevention: true,
            postfix: "",
            imageUploader: {
            brandingHtml: "Powered by u003ca class="icon-imgur-white" href="https://imgur.com/"u003eu003c/au003e",
            contentPolicyHtml: "User contributions licensed under u003ca href="https://creativecommons.org/licenses/by-sa/3.0/"u003ecc by-sa 3.0 with attribution requiredu003c/au003e u003ca href="https://stackoverflow.com/legal/content-policy"u003e(content policy)u003c/au003e",
            allowUrls: true
            },
            noCode: true, onDemand: true,
            discardSelector: ".discard-answer"
            ,immediatelyShowMarkdownHelp:true
            });


            }
            });














            draft saved

            draft discarded


















            StackExchange.ready(
            function () {
            StackExchange.openid.initPostLogin('.new-post-login', 'https%3a%2f%2fmath.stackexchange.com%2fquestions%2f3040217%2fsome-confusion-about-supremum-and-infimum%23new-answer', 'question_page');
            }
            );

            Post as a guest















            Required, but never shown

























            1 Answer
            1






            active

            oldest

            votes








            1 Answer
            1






            active

            oldest

            votes









            active

            oldest

            votes






            active

            oldest

            votes









            6












            $begingroup$

            Of course the supremum is always greater than or equal to the infimum. However, if one has mixtures of $limsup$ and $liminf$, it is indecisive which one is greater. For example, let $$
            a_{mn} = (-1)^n (1-frac{1}{2^m}).
            $$
            Then we have
            $$
            1=liminf_{mtoinfty}limsup_{ntoinfty}a_{mn} >limsup_{mtoinfty}liminf_{ntoinfty}a_{mn} =-1
            $$
            while
            $$
            -1=liminf_{ntoinfty}limsup_{mtoinfty}a_{mn}<limsup_{ntoinfty}liminf_{mtoinfty}a_{mn}=1.
            $$






            share|cite|improve this answer









            $endgroup$


















              6












              $begingroup$

              Of course the supremum is always greater than or equal to the infimum. However, if one has mixtures of $limsup$ and $liminf$, it is indecisive which one is greater. For example, let $$
              a_{mn} = (-1)^n (1-frac{1}{2^m}).
              $$
              Then we have
              $$
              1=liminf_{mtoinfty}limsup_{ntoinfty}a_{mn} >limsup_{mtoinfty}liminf_{ntoinfty}a_{mn} =-1
              $$
              while
              $$
              -1=liminf_{ntoinfty}limsup_{mtoinfty}a_{mn}<limsup_{ntoinfty}liminf_{mtoinfty}a_{mn}=1.
              $$






              share|cite|improve this answer









              $endgroup$
















                6












                6








                6





                $begingroup$

                Of course the supremum is always greater than or equal to the infimum. However, if one has mixtures of $limsup$ and $liminf$, it is indecisive which one is greater. For example, let $$
                a_{mn} = (-1)^n (1-frac{1}{2^m}).
                $$
                Then we have
                $$
                1=liminf_{mtoinfty}limsup_{ntoinfty}a_{mn} >limsup_{mtoinfty}liminf_{ntoinfty}a_{mn} =-1
                $$
                while
                $$
                -1=liminf_{ntoinfty}limsup_{mtoinfty}a_{mn}<limsup_{ntoinfty}liminf_{mtoinfty}a_{mn}=1.
                $$






                share|cite|improve this answer









                $endgroup$



                Of course the supremum is always greater than or equal to the infimum. However, if one has mixtures of $limsup$ and $liminf$, it is indecisive which one is greater. For example, let $$
                a_{mn} = (-1)^n (1-frac{1}{2^m}).
                $$
                Then we have
                $$
                1=liminf_{mtoinfty}limsup_{ntoinfty}a_{mn} >limsup_{mtoinfty}liminf_{ntoinfty}a_{mn} =-1
                $$
                while
                $$
                -1=liminf_{ntoinfty}limsup_{mtoinfty}a_{mn}<limsup_{ntoinfty}liminf_{mtoinfty}a_{mn}=1.
                $$







                share|cite|improve this answer












                share|cite|improve this answer



                share|cite|improve this answer










                answered Dec 15 '18 at 7:11









                SongSong

                14.1k1633




                14.1k1633






























                    draft saved

                    draft discarded




















































                    Thanks for contributing an answer to Mathematics Stack Exchange!


                    • Please be sure to answer the question. Provide details and share your research!

                    But avoid



                    • Asking for help, clarification, or responding to other answers.

                    • Making statements based on opinion; back them up with references or personal experience.


                    Use MathJax to format equations. MathJax reference.


                    To learn more, see our tips on writing great answers.




                    draft saved


                    draft discarded














                    StackExchange.ready(
                    function () {
                    StackExchange.openid.initPostLogin('.new-post-login', 'https%3a%2f%2fmath.stackexchange.com%2fquestions%2f3040217%2fsome-confusion-about-supremum-and-infimum%23new-answer', 'question_page');
                    }
                    );

                    Post as a guest















                    Required, but never shown





















































                    Required, but never shown














                    Required, but never shown












                    Required, but never shown







                    Required, but never shown

































                    Required, but never shown














                    Required, but never shown












                    Required, but never shown







                    Required, but never shown







                    Popular posts from this blog

                    How do I know what Microsoft account the skydrive app is syncing to?

                    Grease: Live!

                    When does type information flow backwards in C++?